Triple integral using cylindrical coordinates

Click For Summary
The discussion focuses on solving a triple integral using cylindrical coordinates, specifically for the region defined by a sphere and a paraboloid. The user successfully set up the integral in cylindrical coordinates but encountered difficulties during the integration process, particularly with the complexity of the resulting equations. They attempted various substitution methods, including u-substitution and trigonometric substitution, but struggled with the presence of multiple differential variables and the boundaries of integration. The user seeks advice on techniques or strategies to simplify the calculations and clarify the integration process. Overall, the conversation highlights the challenges of evaluating complex integrals in multiple coordinate systems.
Draconifors
Messages
17
Reaction score
0

Homework Statement


The first part of the question was to describe E the region within the sphere ##x^2 + y^2 + z^2 = 16## and above the paraboloid ##z=\frac{1}{6} (x^2+y^2)## using the three different coordinate systems.

For cartesian, I found ##4* \int_{0}^{\sqrt{12}} \int_{0}^{12-x^2} \int_{\frac{1}{6} (x^2 + y^2)}^{\sqrt{16-x^2-y^2}} 1 dz dy dx## by splitting the domain into quarters because I found it easier.
For cylindrical I got ##\int_{0}^{2\pi} \int_{0}^{\sqrt{12}} \int_{\frac{r^2}{6}}^{\sqrt{16-r^2}} r dz dr d\theta##.
Spherical gave me ##\int_{0}^{2\pi} \int_{0}^{\frac{\pi}{3}} \int_{0}^{4} \rho^2 sin(\phi) d\rho d\phi d\theta + \int_{0}^{2\pi} \int_{\frac{\pi}{3}}^{\frac{\pi}{2}} \int_{0}^{\frac{6 cos(\phi)}{sin^2(\phi)}} \rho^2 sin(\phi) d\rho d\phi d\theta## by splitting the domain into two with respect to ##\rho##'s boundaries.
(I'm including these mostly as a reference point, without my calculations, but I can go through my calculations if necessary.)

The second part of the question -- the one I'm having trouble with -- is then computing ##\iiint \limits_E (x^2+y^2)^2 dV## in my coordinate system of choice.

Homework Equations



##\cos^2\theta = 1 - \sin^2\theta ##

The Attempt at a Solution


[/B]
So to compute ##\iiint \limits_E (x^2+y^2)^2 dV##, I chose cylindrical because it reduced the inside of the integral to ##r^5##. However, I then get stuck after I integrate with respect to z, because I get $$\int_{0}^{2\pi} \int_{0}^{\sqrt{12}} \left(r^5 * \sqrt{16-r^2} - \frac{r^7}{6} \right) dr d\theta$$.

I first tried u-sub, which doesn't work on the equation as-is. I also attempted trig sub, but I don't know if it's simply because I'm out of practice or whether it's not the system to use, but I can't finish my calculations because of the complexity of the equations I remain with.

I used trig sub with ##r = 4\sin\theta## , ##dr = 4\cos\theta = \sqrt{16-r^2}##.
This gave me $$\int_{0}^{2\pi} \int_{0}^{\sqrt{12}} \left[ (4\sin\theta)^5 * \sqrt{16-16\sin^2\theta} \right) d\theta dr d\theta$$ at which point I confused myself by the presence of the two ## d\theta ##.
Even simply calculating the indefinite integral of the above, I ended up with ##-16384 \left[\frac{u^3}{3} - \frac{2u^5}{5} + \frac{u^7}{7} \right] ##, after having used a u-sub with ##u = \cos\theta## and ##du = -\sin\theta d\theta##. Considering that ##u = \sqrt{1 - (\frac{r}{4})^2}##, I don't see how I'll be able to then integrate with respect to ##\theta##.

I can put up all my calculations if necessary, but what I really want is a technique or strategy hint so that I can solve this, because, from where I'm standing, the boundaries for the three coordinate systems are quite hard to compute with respect to the first integral (i.e. z and ##\rho##).
 
Physics news on Phys.org
Draconifors said:
at which point I confused myself by the presence of the two dθdθ d\theta .
You cannot express ##r## as a function of ##\theta##. You must choose a name for your substitution variable that does not already exist or you will just end up with nonsense.
 
Question: A clock's minute hand has length 4 and its hour hand has length 3. What is the distance between the tips at the moment when it is increasing most rapidly?(Putnam Exam Question) Answer: Making assumption that both the hands moves at constant angular velocities, the answer is ## \sqrt{7} .## But don't you think this assumption is somewhat doubtful and wrong?

Similar threads

  • · Replies 2 ·
Replies
2
Views
2K
  • · Replies 19 ·
Replies
19
Views
2K
  • · Replies 4 ·
Replies
4
Views
2K
  • · Replies 4 ·
Replies
4
Views
1K
  • · Replies 7 ·
Replies
7
Views
2K
Replies
3
Views
2K
  • · Replies 9 ·
Replies
9
Views
1K
  • · Replies 5 ·
Replies
5
Views
2K
  • · Replies 7 ·
Replies
7
Views
5K
  • · Replies 14 ·
Replies
14
Views
2K